2013
DOI: 10.1090/s0025-5718-2013-02733-0
|View full text |Cite
|
Sign up to set email alerts
|

Every odd number greater than $1$ is the sum of at most five primes

Abstract: We prove that every odd number N greater than 1 can be expressed as the sum of at most five primes, improving the result of Ramaré that every even natural number can be expressed as the sum of at most six primes. We follow the circle method of Hardy-Littlewood and Vinogradov, together with Vaughan's identity; our additional techniques, which may be of interest for other Goldbach-type problems, include the use of smoothed exponential sums and optimisation of the Vaughan identity parameters to save or reduce som… Show more

Help me understand this report

Search citation statements

Order By: Relevance

Paper Sections

Select...
1

Citation Types

1
29
0

Year Published

2013
2013
2020
2020

Publication Types

Select...
5
2
1

Relationship

0
8

Authors

Journals

citations
Cited by 24 publications
(30 citation statements)
references
References 39 publications
1
29
0
Order By: Relevance
“…For instance, a Theorem of Tao (see[20]) states that every natural number greater than 1 is the sum of at most five prime numbers.…”
mentioning
confidence: 99%
“…For instance, a Theorem of Tao (see[20]) states that every natural number greater than 1 is the sum of at most five prime numbers.…”
mentioning
confidence: 99%
“…It is known that up to a given number x at most O(x 0.879 ) even integers do not have a Goldbach partition [30], and that every large enough even number is the sum of a prime and the product of at most two primes [24]. Furthermore, according to [48], every odd number greater that one is the sum of at most five primes. As described in Table 1, over a time span of more than a century the even Goldbach conjecture was confirmed to be true up to ever-increasing upper limits.…”
mentioning
confidence: 99%
“…Using the Hardy-Littlewood circle method, Vinogradov [24] established his famous theorem that every sufficiently large odd integer is the sum of three prime numbers. Effective versions of Vinogradov's theorem have been given by several authors (see [10,17,23] and references therein), but for the purposes of the present paper we require only the following extension of Vinogradov's theorem, which is due to Haselgrove [9, Theorem A]. 4.4.…”
Section: P Tmentioning
confidence: 99%